You are on page 1of 112

IMPORTANT

1. Keep your mics on MUTE.


2. Keep your camera ON.
3. Keep your books/notebooks and pens/pencils near you.
4. Keep a bottle of water near you so that you don’t have to get up in
the middle of the session.
5. Be responsive during the class ☺
6. If you face any internet connectivity issues, please log out and log in
again.
7. This class is being recorded. The recording will be available on your
www.jamboree.online portal.
8. For all queries/concerns, please email at
support@jamboreeeducation.com
Digital SAT
SESSION 6
TEST DISCUSSION

© Jamboree Education Pvt Ltd


Cryptozoology is the study of still unknown animals
or of creatures, like the Loch Ness Monster, whose
existence hasn't been proven. Cryptozoology,
however, doesn't just refer to the discovery of animals
that may exist only in the minds of the
overimaginative. It also refers to animals that may
live in areas of the world that are so remote or parts
of the ocean that are so deep that no one has
discovered them. In fact, there is a whole host of
animals, called cryptids that reputable scientists
believe may one day be discovered. A great example
of a cryptid is the coelacanth. This fish was thought
to have been extinct for 70 million years, but it was
discovered in the Indian Ocean in 1938.

As used in the text, what does the word “host” most


nearly mean?

A) Network.
B) Unit.
C) Den.
D) Multitude.

© Jamboree Education Pvt. Ltd.


Cryptozoology is the study of still unknown animals
or of creatures, like the Loch Ness Monster, whose
existence hasn't been proven. Cryptozoology,
however, doesn't just refer to the discovery of animals
that may exist only in the minds of the
overimaginative. It also refers to animals that may
live in areas of the world that are so remote or parts
of the ocean that are so deep that no one has
discovered them. In fact, there is a whole host of
animals, called cryptids that reputable scientists
believe may one day be discovered. A great example
of a cryptid is the coelacanth. This fish was thought
to have been extinct for 70 million years, but it was
discovered in the Indian Ocean in 1938.

As used in the text, what does the word “host” most


nearly mean?

A) Network.
B) Unit.
C) Den.
D) Multitude.

© Jamboree Education Pvt. Ltd.


For thousands of years, money, whether paper or
coins, was a tangible object that people could keep in
their pockets or purses or safely tucked under their
mattresses. The late 20th century saw the birth of
intangible wealth, from e-money to Bowie Bonds.
Bowie Bonds are named after the rock musician,
David Bowie, who in 1997 used his future music as
security for bonds. The bonds were the brainchild of
banker David Pullman. Pullman's innovation caught
fire, and many well-known entertainers, such as
Ashford & Simpson, James Brown, and the Isley
Brothers followed in Bowie's pioneering footsteps.
Today asset-backed securities are becoming the norm.

As used in the text, what does the phrase “caught


fire” most nearly mean?

A) Was destroyed.
B) Sunk into oblivion.
C) Was highly publicized.
D) Became popular.

© Jamboree Education Pvt. Ltd.


For thousands of years, money, whether paper or
coins, was a tangible object that people could keep in
their pockets or purses or safely tucked under their
mattresses. The late 20th century saw the birth of
intangible wealth, from e-money to Bowie Bonds.
Bowie Bonds are named after the rock musician,
David Bowie, who in 1997 used his future music as
security for bonds. The bonds were the brainchild of
banker David Pullman. Pullman's innovation caught
fire, and many well-known entertainers, such as
Ashford & Simpson, James Brown, and the Isley
Brothers followed in Bowie's pioneering footsteps.
Today asset-backed securities are becoming the norm.

As used in the text, what does the phrase “caught


fire” most nearly mean?

A) Was destroyed.
B) Sunk into oblivion.
C) Was highly publicized.
D) Became popular.

© Jamboree Education Pvt. Ltd.


The order was given to loosen the main skysail,
which is the fifth and highest sail from deck. It was a
very small sail, and from the forecastle looked no
bigger than a cambric pocket-handkerchief. But I
have heard that some ships carry still smaller sails,
above the skysail; called moon-sails, and skyscrapers,
and cloudrakers. But I shall not believe in them till I
see them; a skysail seems high enough in all
conscience; and the idea of anything higher than that,
seems preposterous. Besides, it looks almost like
tempting heaven, to brush the very firmament so, and
almost put the eyes of the stars out; when a flaw of
wind, too, might very soon take the conceit out of
these cloud-defying cloudrakers.

Which choice best states the function of the


underlined sentence in the text as a whole?
A) It highlights the spiritual nature of sailing.
B) It illustrates the dangers involved in climbing the
mast.
C) It describes the constellations in the night sky.
D) It emphasizes the height of the skysail.

© Jamboree Education Pvt. Ltd.


The order was given to loosen the main skysail,
which is the fifth and highest sail from deck. It was a
very small sail, and from the forecastle looked no
bigger than a cambric pocket-handkerchief. But I
have heard that some ships carry still smaller sails,
above the skysail; called moon-sails, and skyscrapers,
and cloudrakers. But I shall not believe in them till I
see them; a skysail seems high enough in all
conscience; and the idea of anything higher than that,
seems preposterous. Besides, it looks almost like
tempting heaven, to brush the very firmament so, and
almost put the eyes of the stars out; when a flaw of
wind, too, might very soon take the conceit out of
these cloud-defying cloudrakers.

Which choice best states the function of the


underlined sentence in the text as a whole?
A) It highlights the spiritual nature of sailing.
B) It illustrates the dangers involved in climbing the
mast.
C) It describes the constellations in the night sky.
D) It emphasizes the height of the skysail.

© Jamboree Education Pvt. Ltd.


In 1948, graduate students Norman Woodland
and Bernard Silver took on a problem that had
troubled retailers for years: how to keep track
of store inventories. Inspired by the dots and
dashes of Morse code, __________ created a
system of lines that could encode data. Called a
symbology, the pattern created by the spacing
and widths of the lines encodes information by
representing different characters.

Which choice completes the text with the


most logical transition?
A) however, Woodland and Silver
B) in other words, Woodland and Silver
C) consequently, Woodland and Silver
D) Woodland and Silver

© Jamboree Education Pvt. Ltd.


In 1948, graduate students Norman Woodland
and Bernard Silver took on a problem that had
troubled retailers for years: how to keep track
of store inventories. Inspired by the dots and
dashes of Morse code, __________ created a
system of lines that could encode data. Called a
symbology, the pattern created by the spacing
and widths of the lines encodes information by
representing different characters.

Which choice completes the text with the


most logical transition?
A) however, Woodland and Silver
B) in other words, Woodland and Silver
C) consequently, Woodland and Silver
D) Woodland and Silver

© Jamboree Education Pvt. Ltd.


I had received my first quarter’s wages, and was
returning to my lodgings, possessed heart and soul
with the pleasant feeling that the master who had paid
me grudged every penny of that hard-earned pittance.
Thoughts, not varied but strong, occupied my mind;
two voices spoke within me; again and again, they
uttered the same monotonous phrases. One said:
“William, your life is intolerable.” The other: “What
can you do to alter it?” I walked fast, for it was a
cold, frosty night in January; as I approached my
lodgings, I turned from a general view of my affairs
to the particular speculation as to whether my fire
would be out; looking towards the window of my
sitting-room, I saw no cheering red gleam.

Which of the following is the main concern of the


text?
A) The text is a description of William’s dismal,
cold lodgings.
B) The text is only concerned with the harsh attitude
that William’s employer has towards him.
C) The text describes overall dissatisfaction William
has for his life.
D) The text is mostly concerned with harsh and cold
weather William has to bear.

© Jamboree Education Pvt. Ltd.


I had received my first quarter’s wages, and was
returning to my lodgings, possessed heart and soul
with the pleasant feeling that the master who had paid
me grudged every penny of that hard-earned pittance.
Thoughts, not varied but strong, occupied my mind;
two voices spoke within me; again and again, they
uttered the same monotonous phrases. One said:
“William, your life is intolerable.” The other: “What
can you do to alter it?” I walked fast, for it was a
cold, frosty night in January; as I approached my
lodgings, I turned from a general view of my affairs
to the particular speculation as to whether my fire
would be out; looking towards the window of my
sitting-room, I saw no cheering red gleam.

Which of the following is the main concern of the


text?
A) The text is a description of William’s dismal,
cold lodgings.
B) The text is only concerned with the harsh attitude
that William’s employer has towards him.
C) The text describes overall dissatisfaction William
has for his life.
D) The text is mostly concerned with harsh and cold
weather William has to bear.

© Jamboree Education Pvt. Ltd.


For people with synesthesia, a condition in which the
senses get mixed up, the number five may be red.
One explanation for this sensory crossover is that
synesthetes, people with synesthesia, are simply
experiencing childhood memories and associations.
Maybe the person played with refrigerator magnets as
a child and the number five was red. Another
explanation is that synesthetes are experiencing the
result of some kind of cross wiring between regions
of the brain. In fact, we have now identified where in
the brain such cross wiring might occur. We have also
determined that synesthesia can occur if the wiring is
fine but the balance of chemicals that control brain
activity is skewed.

Which choice best states the primary purpose of the


text?
A) It explores possible explanations for a
phenomenon.
B) It compares two somewhat related phenomena.
C) It presents a theory for a phenomenon and then
critique that theory.
D) It traces the evolution of an intriguing
phenomenon.
© Jamboree Education Pvt. Ltd.
For people with synesthesia, a condition in which the
senses get mixed up, the number five may be red.
One explanation for this sensory crossover is that
synesthetes, people with synesthesia, are simply
experiencing childhood memories and associations.
Maybe the person played with refrigerator magnets as
a child and the number five was red. Another
explanation is that synesthetes are experiencing the
result of some kind of cross wiring between regions
of the brain. In fact, we have now identified where in
the brain such cross wiring might occur. We have also
determined that synesthesia can occur if the wiring is
fine but the balance of chemicals that control brain
activity is skewed.

Which choice best states the primary purpose of the


text?
A) It explores possible explanations for a
phenomenon.
B) It compares two somewhat related phenomena.
C) It presents a theory for a phenomenon and then
critique that theory.
D) It traces the evolution of an intriguing
phenomenon.
© Jamboree Education Pvt. Ltd.
In an experiment, students at the University of Southampton,
England, were paid to enter data electronically. They were
given a small base reward for each entry and later offered the
chance to either earn additional bonuses for themselves or
provide a donation to charity. While those who were initially
highly productive showed mostly decreases in their
productivity, the other workers had substantial productivity
gains. Among all workers, the study showed productivity gains
of 17 percent to 34 percent based on the different reward
structures, but the largest effect recorded in the study – a 73
percent increase in productivity – occurred when workers with
low initial productivity were _________________

Which choice most effectively uses data from the table to


complete the text?
A) given a bonus of 10 pence as a personal incentive.
B) given 2.5 pence as a personal incentive with the option to
donate 5 pence.
C) given 2.5 pence as a personal incentive with the option to
donate 10 pence.
D) allowed to choose how much of a 10-pence incentive to
donate to charity.

© Jamboree Education Pvt. Ltd.


In an experiment, students at the University of Southampton,
England, were paid to enter data electronically. They were
given a small base reward for each entry and later offered the
chance to either earn additional bonuses for themselves or
provide a donation to charity. While those who were initially
highly productive showed mostly decreases in their
productivity, the other workers had substantial productivity
gains. Among all workers, the study showed productivity gains
of 17 percent to 34 percent based on the different reward
structures, but the largest effect recorded in the study – a 73
percent increase in productivity – occurred when workers with
low initial productivity were _________________

Which choice most effectively uses data from the table to


complete the text?
A) given a bonus of 10 pence as a personal incentive.
B) given 2.5 pence as a personal incentive with the option to
donate 5 pence.
C) given 2.5 pence as a personal incentive with the option to
donate 10 pence.
D) allowed to choose how much of a 10-pence incentive to
donate to charity.

© Jamboree Education Pvt. Ltd.


Last year the city of Somerton discontinued its train
service because low ridership caused total fares
collected to be substantially lower than the cost of
operating the service. However, the number of people
living in communities near the Somerton train line
has increased significantly this year. Therefore,
potential ridership must also have increased, and
some analysts claim that there is a good chance that
restoring the Somerton train service this year would
not result in serious deficits.

Which of the following, if true, would most strengthen


the analysts’ claim?
A) Once rail service along a given route has been
discontinued, there are substantial start-up costs
associated with restoring service along that route.
B) Most potential Somerton riders are unwilling to pay
more for the train service than riders paid last year.
C) Very few of the Somerton line's former daily
commuters find the means of transportation that they
now use as satisfactory as the train service had been.
D) Several retail businesses that once each had a single
store, located in the Somerton business district near
the train station, now also have branches that are
conveniently located in communities near Somerton.

© Jamboree Education Pvt. Ltd.


Last year the city of Somerton discontinued its train
service because low ridership caused total fares
collected to be substantially lower than the cost of
operating the service. However, the number of people
living in communities near the Somerton train line
has increased significantly this year. Therefore,
potential ridership must also have increased, and
some analysts claim that there is a good chance that
restoring the Somerton train service this year would
not result in serious deficits.

Which of the following, if true, would most strengthen


the analysts’ claim?
A) Once rail service along a given route has been
discontinued, there are substantial start-up costs
associated with restoring service along that route.
B) Most potential Somerton riders are unwilling to pay
more for the train service than riders paid last year.
C) Very few of the Somerton line's former daily
commuters find the means of transportation that they
now use as satisfactory as the train service had been.
D) Several retail businesses that once each had a single
store, located in the Somerton business district near
the train station, now also have branches that are
conveniently located in communities near Somerton.

© Jamboree Education Pvt. Ltd.


When Amelia was growing up in the 1950s, space
travel and artificial intelligence existed only in
science fiction stories. A half century later, at NASA,
Amelia witnessed the realization of both. During a
visit to a virtual airport control center,
___________________courtesy of a video feed
provided by the Mars Global Surveyor satellite. At
the Jet Propulsion Laboratory in Pasadena, California,
she was introduced to robots that function
autonomously through control-and-sensor-processing
software.

Which choice completes the text so that it


conforms to the conventions of Standard English?
A) it was the panoramic images of the red planet that
Amelia viewed,
B) Amelia viewed panoramic images of the red
planet,
C) panoramic images of the red planet were shown
to Amelia,
D) panoramic images, which were of the red planet,
were viewed by Amelia,

© Jamboree Education Pvt. Ltd.


When Amelia was growing up in the 1950s, space
travel and artificial intelligence existed only in
science fiction stories. A half century later, at NASA,
Amelia witnessed the realization of both. During a
visit to a virtual airport control center,
___________________courtesy of a video feed
provided by the Mars Global Surveyor satellite. At
the Jet Propulsion Laboratory in Pasadena, California,
she was introduced to robots that function
autonomously through control-and-sensor-processing
software.

Which choice completes the text so that it


conforms to the conventions of Standard English?
A) it was the panoramic images of the red planet that
Amelia viewed,
B) Amelia viewed panoramic images of the red
planet,
C) panoramic images of the red planet were shown
to Amelia,
D) panoramic images, which were of the red planet,
were viewed by Amelia,

© Jamboree Education Pvt. Ltd.


This all started on a Saturday morning in May,
one of those warm spring days that smell like
clean linen. Delia had gone to the supermarket
to shop for the week's meals. She was standing
in the produce section, languidly choosing a
bunch of celery'. Grocery stores always made
her reflective. Why was it, she was wondering
that celery was not called "corduroy plant"?
That would be much more colorful. And garlic
bulbs should be "moneybags," because their
shape reminded her of the sacks of gold coins
in folktales.

Which choice best states the function of the


underlined sentence in the text as a whole?
A) It explains a course of action.
B) It evokes a particular sensation.
C) It describes an unexpected development.
D) It points out a pressing obligation.

© Jamboree Education Pvt. Ltd.


This all started on a Saturday morning in May,
one of those warm spring days that smell like
clean linen. Delia had gone to the supermarket
to shop for the week's meals. She was standing
in the produce section, languidly choosing a
bunch of celery'. Grocery stores always made
her reflective. Why was it, she was wondering
that celery was not called "corduroy plant"?
That would be much more colorful. And garlic
bulbs should be "moneybags," because their
shape reminded her of the sacks of gold coins
in folktales.

Which choice best states the function of the


underlined sentence in the text as a whole?
A) It explains a course of action.
B) It evokes a particular sensation.
C) It describes an unexpected development.
D) It points out a pressing obligation.

© Jamboree Education Pvt. Ltd.


Some critics believe that the frequent use of
repetition in Native American ceremonial texts was a
result of their oral nature and helped make the works
easy to remember. Native American scholar Paula
Gunn Allen argues that this factor must be peripheral,
however, because people in societies without writing
traditionally have had more finely developed
memories than do people who use writing. Native
American children learned early to remember
complicated instructions and long stories by heart.
For a person who couldn’t run to a bookshelf to look
up information, reliance on memory became very
important in everyday life. Such a highly developed
everyday memory is not likely to fail on ceremonial
occasions.

Which choice best states the main purpose of the


text?

A) It describes a process.
B) It refutes a claim.
C) It advocates a practice.
D) It reveals a problem.

© Jamboree Education Pvt. Ltd.


Some critics believe that the frequent use of
repetition in Native American ceremonial texts was a
result of their oral nature and helped make the works
easy to remember. Native American scholar Paula
Gunn Allen argues that this factor must be peripheral,
however, because people in societies without writing
traditionally have had more finely developed
memories than do people who use writing. Native
American children learned early to remember
complicated instructions and long stories by heart.
For a person who couldn’t run to a bookshelf to look
up information, reliance on memory became very
important in everyday life. Such a highly developed
everyday memory is not likely to fail on ceremonial
occasions.

Which choice best states the main purpose of the


text?

A) It describes a process.
B) It refutes a claim.
C) It advocates a practice.
D) It reveals a problem.

© Jamboree Education Pvt. Ltd.


Aphasia, an impairment of the capacity to use language,
often occurs when a stroke damages the left half of the brain.
Many people with stroke-related aphasia recover at least
some capacity to use language within a year. Researchers
attempting to explain such recoveries propose that the right
side of the brain, which is not usually the major language
center, develops its latent language capabilities to
compensate for the damage to the left side.

Which of the following, if true, would most strengthen the


researchers’ explanation?
A) In a study of local brain activity in people performing a
language task, people with stroke related aphasia
showed higher activity levels in the right half of the
brain than people who did not have aphasia.
B) A blow to the head injuring the left half of the brain can
result in impairment of the capacity to use language
indistinguishable from that produced by a stroke.
C) A stroke that damages the left half of the brain often
causes physical impairments of the right side of the
body that lessen over time.
D) Studies of numerous people with aphasia have indicated
that the functions that govern language production and
those that govern language comprehension are located in
separate areas of the brain.

© Jamboree Education Pvt. Ltd.


Aphasia, an impairment of the capacity to use language,
often occurs when a stroke damages the left half of the brain.
Many people with stroke-related aphasia recover at least
some capacity to use language within a year. Researchers
attempting to explain such recoveries propose that the right
side of the brain, which is not usually the major language
center, develops its latent language capabilities to
compensate for the damage to the left side.

Which of the following, if true, would most strengthen the


researchers’ explanation?
A) In a study of local brain activity in people performing a
language task, people with stroke related aphasia
showed higher activity levels in the right half of the
brain than people who did not have aphasia.
B) A blow to the head injuring the left half of the brain can
result in impairment of the capacity to use language
indistinguishable from that produced by a stroke.
C) A stroke that damages the left half of the brain often
causes physical impairments of the right side of the
body that lessen over time.
D) Studies of numerous people with aphasia have indicated
that the functions that govern language production and
those that govern language comprehension are located in
separate areas of the brain.

© Jamboree Education Pvt. Ltd.


One way to increase the output of wind
farms is to create new turbines that have
longer rotor blades than previous models
had. Data from the European Wind Energy
Association show a consistent trend in this
direction since the 1980s, with turbine rotor
diameters increasing every few years. In the
period between 2010 and 2015, the rotor
diameter of new turbines reached 100
meters, several time that of turbines in the
1980s. As rotor blades have gotten longer,
power generation has gone up dramatically:
a modern turbine can generate up to 5,000
kilowatts of wind power, compared with
about ___________________________

Which choice most effectively uses data


from the graph to complete the text?
A) 750 kilowatts for one from the 2000s.
B) 300 kilowatts for one from 1995-2000.
C) 75 kilowatts for one from the 1980s.
D) 1,500 kilowatts for one from 2005-2010.

© Jamboree Education Pvt. Ltd.


One way to increase the output of wind
farms is to create new turbines that have
longer rotor blades than previous models
had. Data from the European Wind Energy
Association show a consistent trend in this
direction since the 1980s, with turbine rotor
diameters increasing every few years. In the
period between 2010 and 2015, the rotor
diameter of new turbines reached 100
meters, several time that of turbines in the
1980s. As rotor blades have gotten longer,
power generation has gone up dramatically:
a modern turbine can generate up to 5,000
kilowatts of wind power, compared with
about ___________________________

Which choice most effectively uses data


from the graph to complete the text?
A) 750 kilowatts for one from the 2000s.
B) 300 kilowatts for one from 1995-2000.
C) 75 kilowatts for one from the 1980s.
D) 1,500 kilowatts for one from 2005-2010.

© Jamboree Education Pvt. Ltd.


Text 1
The French Revolution reverberated around the world when it Based on the texts, how would the
happened – a telling blow was struck for the people's rights, and author of Text 1 regard the social and
a step was taken toward true democracy that could not be taken economic pressures mentioned in Text
back. We are indebted to renegade artists for lighting the match 2?
that set that blaze. In the eighteenth century, it was unheard of A) As noncontributing factors in the
for commoners to speak out against the king. Several guerrilla French Revolution.
theater groups performing outside the castle, however, began B) As making the French Revolution
doing shocking things; they mocked the royalty, burned them in less likely.
effigy, and drenched them in scorn. The performances gave the C) As being insufficient to incite the
downtrodden commoners the courage and license they required French Revolution on their own.
to form a body to rise against the monarchy. D) As being the most important
Text 2 contributing factors to the French
Revolution.
The French Revolution was a powerful and inevitable
occurrence. Social and economic pressures gave the peasants of
France literally no choice but to fight against their oppressive
government. Much has been made recently of the impact of a
few ragtag theater groups outside the palace; while they
probably did a good job of reflecting the growing discontent of
the people, it is certain that a family's need to eat was a more
compelling argument for revolution than a funny play that
Father saw on the way home from the fields. These plays
provided a mirror, not a catalyst, for the mood of the populace.

© Jamboree Education Pvt. Ltd.


Text 1
The French Revolution reverberated around the world when it Based on the texts, how would the
happened – a telling blow was struck for the people's rights, and author of Text 1 regard the social and
a step was taken toward true democracy that could not be taken economic pressures mentioned in Text
back. We are indebted to renegade artists for lighting the match 2?
that set that blaze. In the eighteenth century, it was unheard of A) As noncontributing factors in the
for commoners to speak out against the king. Several guerrilla French Revolution.
theater groups performing outside the castle, however, began B) As making the French Revolution
doing shocking things; they mocked the royalty, burned them in less likely.
effigy, and drenched them in scorn. The performances gave the C) As being insufficient to incite the
downtrodden commoners the courage and license they required French Revolution on their own.
to form a body to rise against the monarchy. D) As being the most important
Text 2 contributing factors to the French
Revolution.
The French Revolution was a powerful and inevitable
occurrence. Social and economic pressures gave the peasants of
France literally no choice but to fight against their oppressive
government. Much has been made recently of the impact of a
few ragtag theater groups outside the palace; while they
probably did a good job of reflecting the growing discontent of
the people, it is certain that a family's need to eat was a more
compelling argument for revolution than a funny play that
Father saw on the way home from the fields. These plays
provided a mirror, not a catalyst, for the mood of the populace.

© Jamboree Education Pvt. Ltd.


Mrs. Quabarl, to use a colloquial expression, was knocked
off her perch. She was one of those imperfectly self-assured
individuals who are magnificent and autocratic as long as
they are not seriously opposed. The least show of
unexpected resistance goes a long way towards rendering
them cowed and apologetic. When the new governess failed
to express wondering admiration of the large newly-
purchased and expensive car, and lightly alluded to the
superior advantages of one or two makes which had just
been put on the market, the discomfiture of her patroness
became almost abject. Her feelings were those which might
have animated a general of ancient warfaring days, on
beholding his heaviest battle-elephant ignominiously driven
off the field by slingers and javelin throwers.

Which choice best states the central idea of the text?


A) Description of Mrs. Quabarl’s new, expensive car and
the new governess’ indifference towards it.
B) Description of Mrs. Quabarl’s character and her
feelings towards the new governess.
C) Description of Mrs. Quabarl’s autocratic, warfaring
nature.
D) Description of Mrs. Quabarl’s experience in the war.
© Jamboree Education Pvt. Ltd.
Mrs. Quabarl, to use a colloquial expression, was knocked
off her perch. She was one of those imperfectly self-assured
individuals who are magnificent and autocratic as long as
they are not seriously opposed. The least show of
unexpected resistance goes a long way towards rendering
them cowed and apologetic. When the new governess failed
to express wondering admiration of the large newly-
purchased and expensive car, and lightly alluded to the
superior advantages of one or two makes which had just
been put on the market, the discomfiture of her patroness
became almost abject. Her feelings were those which might
have animated a general of ancient warfaring days, on
beholding his heaviest battle-elephant ignominiously driven
off the field by slingers and javelin throwers.

Which choice best states the central idea of the text?


A) Description of Mrs. Quabarl’s new, expensive car and
the new governess’ indifference towards it.
B) Description of Mrs. Quabarl’s character and her
feelings towards the new governess.
C) Description of Mrs. Quabarl’s autocratic, warfaring
nature.
D) Description of Mrs. Quabarl’s experience in the war.
© Jamboree Education Pvt. Ltd.
The concept of free trade is simple: if Company A can
produce and distribute a product more efficiently and
at a higher quality than Company B, it should be
allowed to do so, and to charge any price for it that
free consumers are willing to pay. Although Company
B would likely suffer as a result, humanity would
benefit from freer and cheaper access to high-quality
goods. Sometimes free trade works nicely, as when
Company A is in the United States and Company B is
in India. Then, agreements are signed to “open up”
India to the cheaper goods made by Company A, even
if doing so crushes Company B because, we say,
consumers have a right to cheap, high-quality goods.
But if Company A were in India and Company B
were in the United States, the story would likely be
very different.

Which choice best states the main purpose of the


text?

A) It illustrates a debate.
B) It provides a statistical analysis.
C) It explains a concept.
D) It gives historical background.
© Jamboree Education Pvt. Ltd.
The concept of free trade is simple: if Company A can
produce and distribute a product more efficiently and
at a higher quality than Company B, it should be
allowed to do so, and to charge any price for it that
free consumers are willing to pay. Although Company
B would likely suffer as a result, humanity would
benefit from freer and cheaper access to high-quality
goods. Sometimes free trade works nicely, as when
Company A is in the United States and Company B is
in India. Then, agreements are signed to “open up”
India to the cheaper goods made by Company A, even
if doing so crushes Company B because, we say,
consumers have a right to cheap, high-quality goods.
But if Company A were in India and Company B
were in the United States, the story would likely be
very different.

Which choice best states the main purpose of the


text?

A) It illustrates a debate.
B) It provides a statistical analysis.
C) It explains a concept.
D) It gives historical background.
© Jamboree Education Pvt. Ltd.
Working together, __________________which
catalogs and identifies zebras. StripeSpotter translates
the pattern of stripes on a zebra’s side into an
identifier, like a bar code, that can be compared to
other zebra stripe-pattern identifiers that have been
stored in a database.
Which choice completes the text so that it
conforms to the conventions of Standard English?
A) StripeSpotter, which was a software program, was
developed by scientists at the University of
Illinois and Princeton University,
B) a software program called StripeSpotter was
developed by scientists at the University of
Illinois and Princeton University,
C) it was the development of StripeSpotter, which
was a software program, that was achieved by
scientists at the University of Illinois and
Princeton University,
D) scientists at the University of Illinois and
Princeton University developed a software
program called StripeSpotter,

© Jamboree Education Pvt. Ltd.


Working together, __________________which
catalogs and identifies zebras. StripeSpotter translates
the pattern of stripes on a zebra’s side into an
identifier, like a bar code, that can be compared to
other zebra stripe-pattern identifiers that have been
stored in a database.
Which choice completes the text so that it
conforms to the conventions of Standard English?
A) StripeSpotter, which was a software program, was
developed by scientists at the University of
Illinois and Princeton University,
B) a software program called StripeSpotter was
developed by scientists at the University of
Illinois and Princeton University,
C) it was the development of StripeSpotter, which
was a software program, that was achieved by
scientists at the University of Illinois and
Princeton University,
D) scientists at the University of Illinois and
Princeton University developed a software
program called StripeSpotter,

© Jamboree Education Pvt. Ltd.


Cultivating hybrid-teas ______________ a
process of grafting two species of rose together,
but the grafted area remains weak and
susceptible to viruses. Antiques, on the other
hand, are less prone to disease because they are
grown simply by placing cuttings from a parent
plant into nutrient-rich soil.

Which choice completes the text so that it


conforms to the conventions of Standard
English?
A) having involved
B) have involved
C) involves
D) involve

© Jamboree Education Pvt. Ltd.


Cultivating hybrid-teas ______________ a
process of grafting two species of rose together,
but the grafted area remains weak and
susceptible to viruses. Antiques, on the other
hand, are less prone to disease because they are
grown simply by placing cuttings from a parent
plant into nutrient-rich soil.

Which choice completes the text so that it


conforms to the conventions of Standard
English?
A) having involved
B) have involved
C) involves
D) involve

© Jamboree Education Pvt. Ltd.


Text 1
The first Industrial Revolution, which occurred in Great Britain in the latter Which statement best describes a
half of the 18th century, represented a sudden acceleration of technological significant difference between
and economic development that would permeate all levels of British society. the authors' interpretations of
Specifically, the traditional agrarian economy was supplanted by one based population increase and its
on manufacturing and machinery. Very much an urban movement, the relationship to the Industrial
revolution gave rise to a new system of social class, based primarily upon the Revolution?
relationship of the industrial capitalist to the factory worker. These changes
A) Author 1 maintains that a
can be attributed to a number of favorable societal circumstances-including
growing population was not a
an increasing population, which would provide both a larger workforce and
major factor in the revolution;
expanding markets, a strong middle class, and stability in both the political
author 2 maintains that it was a
environment and the monetary system.
prerequisite.
Text 2 B) Author 1 emphasizes that it
was a main cause; author 2
Though the Industrial Revolution certainly saw the transformation of many
claims that it had no effect on
different aspects of British social and economic life, these changes were
the revolution.
primarily effects, not to be mistaken for causes. Undoubtedly, the burgeoning
C) Author 1 states that it was one
population and established political system provided an apt environment for
of the major factors; author 2
revolution. Yet the chief factors were rooted not in broad changes in society
claims that it was important for
but rather in extraordinary technological innovations within a few industries.
setting the stage but was not
Within the smelting industry, for example, the production of new materials,
the main catalyst.
namely iron and steel, would allow for stronger, more complex machinery.
D) Author 1 contends that
Coupled with the invention of Jam es Watt's steam engine in the 1780s, these
population growth was greatest
innovations laid the groundwork for massive technological progress, which
in the more industrialized
would in turn pave the way for those significant social and economic
regions; author 2 maintains
changes.
that population growth was
© Jamboree Education Pvt. Ltd. suppressed in those areas.
Text 1
The first Industrial Revolution, which occurred in Great Britain in the latter Which statement best describes a
half of the 18th century, represented a sudden acceleration of technological significant difference between
and economic development that would permeate all levels of British society. the authors' interpretations of
Specifically, the traditional agrarian economy was supplanted by one based population increase and its
on manufacturing and machinery. Very much an urban movement, the relationship to the Industrial
revolution gave rise to a new system of social class, based primarily upon the Revolution?
relationship of the industrial capitalist to the factory worker. These changes
A) Author 1 maintains that a
can be attributed to a number of favorable societal circumstances-including
growing population was not a
an increasing population, which would provide both a larger workforce and
major factor in the revolution;
expanding markets, a strong middle class, and stability in both the political
author 2 maintains that it was a
environment and the monetary system.
prerequisite.
Text 2 B) Author 1 emphasizes that it
was a main cause; author 2
Though the Industrial Revolution certainly saw the transformation of many
claims that it had no effect on
different aspects of British social and economic life, these changes were
the revolution.
primarily effects, not to be mistaken for causes. Undoubtedly, the burgeoning
C) Author 1 states that it was one
population and established political system provided an apt environment for
of the major factors; author 2
revolution. Yet the chief factors were rooted not in broad changes in society
claims that it was important for
but rather in extraordinary technological innovations within a few industries.
setting the stage but was not
Within the smelting industry, for example, the production of new materials,
the main catalyst.
namely iron and steel, would allow for stronger, more complex machinery.
D) Author 1 contends that
Coupled with the invention of Jam es Watt's steam engine in the 1780s, these
population growth was greatest
innovations laid the groundwork for massive technological progress, which
in the more industrialized
would in turn pave the way for those significant social and economic
regions; author 2 maintains
changes.
that population growth was
© Jamboree Education Pvt. Ltd. suppressed in those areas.
Today, microwaves are used for many purposes: long-distance
telephone calls, television programming, and even communications
between Earth and objects in space. But the microwave is probably
most familiar to us as an energy source for cooking food. Since
their accidental invention fifty years ago, microwave ovens have
become standard equipment in American homes.
___________________ Despite their familiarity, some have
claimed that microwave cooking has a negative effect on human
health.

Which choice completes the text with the most logical transition?
A) The fact that the government allows them to be sold makes it
clear that microwaves are safe for cooking food for human
consumption.
B) Families who use microwave ovens, however, tend not to use
long-distance and television programming services that use
microwaves due to the possible negative impact of these
services on health.
C) Many are unaware that a microwave oven can reduce cooking
time, making it possible for families to engage in other
activities, such as calling faraway friends and watching
television.
D) An estimated 90 percent of American homes have them,
indicating that microwaves are an indispensable tool for most
cooks.
© Jamboree Education Pvt. Ltd.
Today, microwaves are used for many purposes: long-distance
telephone calls, television programming, and even communications
between Earth and objects in space. But the microwave is probably
most familiar to us as an energy source for cooking food. Since
their accidental invention fifty years ago, microwave ovens have
become standard equipment in American homes.
___________________ Despite their familiarity, some have
claimed that microwave cooking has a negative effect on human
health.

Which choice completes the text with the most logical transition?
A) The fact that the government allows them to be sold makes it
clear that microwaves are safe for cooking food for human
consumption.
B) Families who use microwave ovens, however, tend not to use
long-distance and television programming services that use
microwaves due to the possible negative impact of these
services on health.
C) Many are unaware that a microwave oven can reduce cooking
time, making it possible for families to engage in other
activities, such as calling faraway friends and watching
television.
D) An estimated 90 percent of American homes have them,
indicating that microwaves are an indispensable tool for most
cooks.
© Jamboree Education Pvt. Ltd.
The widths of the lines _____________
perfectly to the widths of the zebra’s stripes.
This is the zebra’s “StripeCode,” unique to each
animal in much the same way a fingerprint is
unique to each person. The StripeCode is
logged in the database, where a researcher
uploading a new photograph of a zebra can scan
the stored codes to find a potential match.

Which choice completes the text so that it


conforms to the conventions of Standard
English?
A) corresponds
B) has corresponded
C) is corresponding
D) correspond

© Jamboree Education Pvt. Ltd.


The widths of the lines _____________
perfectly to the widths of the zebra’s stripes.
This is the zebra’s “StripeCode,” unique to each
animal in much the same way a fingerprint is
unique to each person. The StripeCode is
logged in the database, where a researcher
uploading a new photograph of a zebra can scan
the stored codes to find a potential match.

Which choice completes the text so that it


conforms to the conventions of Standard
English?
A) corresponds
B) has corresponded
C) is corresponding
D) correspond

© Jamboree Education Pvt. Ltd.


Although scientists knew of the existence of huge
internal ocean waves in the South China Sea and
beyond, they didn’t know exactly how these waves
formed. To find out, Tom Peacock and a team of
researchers from M.I.T. and Woods Hole
Oceanographic Institution worked with France’s
National Center for Scientific Research using a giant
facility there called the Coriolis Platform. The rotating
platform, about 15 meters (49.2 feet) in diameter,
turns at variable speeds and can simulate Earth’s
rotation. It also has walls, which means scientists can
fill it with water and create accurate, large-scale
simulations of various oceanographic scenarios.

Which choice best states the central claim of the


text?
A) Tom Peacock and his team work at M.I.T. and at
Woods Hole Oceanographic Institution to study
internal waves.
B) Coriolis Platform, which is 15 meters in diameter,
rotates in order to imitate Earth’s rotation.
C) Huge internal waves exist in many parts of the
world, including South China Sea.
D) Tom Peacock and his team’s attempt at
understanding the formation of internal waves.

© Jamboree Education Pvt. Ltd.


Although scientists knew of the existence of huge
internal ocean waves in the South China Sea and
beyond, they didn’t know exactly how these waves
formed. To find out, Tom Peacock and a team of
researchers from M.I.T. and Woods Hole
Oceanographic Institution worked with France’s
National Center for Scientific Research using a giant
facility there called the Coriolis Platform. The rotating
platform, about 15 meters (49.2 feet) in diameter,
turns at variable speeds and can simulate Earth’s
rotation. It also has walls, which means scientists can
fill it with water and create accurate, large-scale
simulations of various oceanographic scenarios.

Which choice best states the central claim of the


text?
A) Tom Peacock and his team work at M.I.T. and at
Woods Hole Oceanographic Institution to study
internal waves.
B) Coriolis Platform, which is 15 meters in diameter,
rotates in order to imitate Earth’s rotation.
C) Huge internal waves exist in many parts of the
world, including South China Sea.
D) Tom Peacock and his team’s attempt at
understanding the formation of internal waves.

© Jamboree Education Pvt. Ltd.


All these years I, like so many others, had stood Which choice best states the function of the
in front of a thousand, two thousand, who- underlined portion from the author of the
knows-how-many thousand Pollocks, de text?
Koonings, Newmans, Nolands, Rothkos,
Rauschenbergs, Judds, Johnses, Olitsis, Louises, A) The author chastises himself for his
Stills, Franz, Klines, Frankehtlers, Kellys, and ignorance.
Frank Stellas, now squinting, now popping the B) The author impugns the readers’
eye sockets open, now drawing back, now
complacency by mocking them.
moving closer—waiting, waiting, forever
waiting for it to come into focus, namely, the C) The author suggests that not all art is
visual reward (for so much effort) which must be intended to be taken seriously.
there, which everyone knew to be there— D) The author pokes fun at artists who are
waiting for something to radiate directly from too somber about their art.
the paintings on these invariably pure white
walls, in this room, in this moment, into my own
optic chiasma. All these years, in short, I had
assumed that in art, if nowhere else, seeing is
believing. Well—how very shortsighted! Now, at
last, on April 28, 1974, I could see. I had gotten
it backward all along. Not “seeing is believing,”
you ninny, but “believing is seeing,” for Modern
Art has become completely literary: the
paintings and other works exist only to illustrate
the text.

© Jamboree Education Pvt. Ltd.


All these years I, like so many others, had stood Which choice best states the function of the
in front of a thousand, two thousand, who- underlined portion from the author of the
knows-how-many thousand Pollocks, de text?
Koonings, Newmans, Nolands, Rothkos,
Rauschenbergs, Judds, Johnses, Olitsis, Louises, A) The author chastises himself for his
Stills, Franz, Klines, Frankehtlers, Kellys, and ignorance.
Frank Stellas, now squinting, now popping the B) The author impugns the readers’
eye sockets open, now drawing back, now
complacency by mocking them.
moving closer—waiting, waiting, forever
waiting for it to come into focus, namely, the C) The author suggests that not all art is
visual reward (for so much effort) which must be intended to be taken seriously.
there, which everyone knew to be there— D) The author pokes fun at artists who are
waiting for something to radiate directly from too somber about their art.
the paintings on these invariably pure white
walls, in this room, in this moment, into my own
optic chiasma. All these years, in short, I had
assumed that in art, if nowhere else, seeing is
believing. Well—how very shortsighted! Now, at
last, on April 28, 1974, I could see. I had gotten
it backward all along. Not “seeing is believing,”
you ninny, but “believing is seeing,” for Modern
Art has become completely literary: the
paintings and other works exist only to illustrate
the text.

© Jamboree Education Pvt. Ltd.


When a major environmental change occurs suddenly, one
possibility is for the pace of evolution to increase. Another
possibility is extinction. The earth has witnessed several mass
extinctions – worldwide ecosystem catastrophes that affect
multiple species. The biggest one divided the era of “ancient
life” (the Paleozoic) from the era of “middle life” (the
Mesozoic). This mass extinction occurred 245 million years ago,
when 4.5 million of the earth’s estimated 5 million species
(mostly invertebrates) were wiped out. The second biggest
extinction, which occurred 65 million years ago, destroyed the
dinosaurs and many other Mesozoic species. One explanation for
the extinction of the dinosaurs is that a massive, long-lasting
cloud of gas and dust arose from the impact of a huge meteorite.
The cloud blocked solar radiation and therefore photosynthesis,
ultimately destroying most plants and the chain of animals that
fed on them.

According to the text, why did the dinosaurs most likely


become extinct?

A) Because they were struck by a large meteorite.


B) Because their food supply was eliminated.
C) Because a more dominant species invaded their environment
and destroyed them.
D) Because the earth’s temperature increased dramatically after
the impact of a meteorite.
© Jamboree Education Pvt. Ltd.
When a major environmental change occurs suddenly, one
possibility is for the pace of evolution to increase. Another
possibility is extinction. The earth has witnessed several mass
extinctions – worldwide ecosystem catastrophes that affect
multiple species. The biggest one divided the era of “ancient
life” (the Paleozoic) from the era of “middle life” (the
Mesozoic). This mass extinction occurred 245 million years ago,
when 4.5 million of the earth’s estimated 5 million species
(mostly invertebrates) were wiped out. The second biggest
extinction, which occurred 65 million years ago, destroyed the
dinosaurs and many other Mesozoic species. One explanation for
the extinction of the dinosaurs is that a massive, long-lasting
cloud of gas and dust arose from the impact of a huge meteorite.
The cloud blocked solar radiation and therefore photosynthesis,
ultimately destroying most plants and the chain of animals that
fed on them.

According to the text, why did the dinosaurs most likely


become extinct?

A) Because they were struck by a large meteorite.


B) Because their food supply was eliminated.
C) Because a more dominant species invaded their environment
and destroyed them.
D) Because the earth’s temperature increased dramatically after
the impact of a meteorite.
© Jamboree Education Pvt. Ltd.
For most of human history the plight of the poor has been
easily excluded from the consciousness of those with the
power to act. Inaction was justified by elaborate theories that
the poor were by nature inferior or happy in their condition or
both. But human society confronts a new reality: pressures of
population and technology on the fragile balance of our
planet’s ecosystem are compelling the rich not only to re-
examine their own lifestyles, but also to take a greater interest
in the lifestyles of the poor whose conditions leave them
dependent for short-term survival on large families and the
destructive use of crop lands, forests, and watersheds—thus
posing a threat to the affluent more certain in its occurrence
and consequences than the threat of armed revolution. Poverty
has become salient to the powerful and the implications are far
reaching.

The text suggests that the poor affect the lives of the
affluent chiefly by doing what of the following?

A) They drain money from the government.


B) They damage the natural environment.
C) They make the wealthy feel guilty for their relative
affluence.
D) They threaten armed revolution.

© Jamboree Education Pvt. Ltd.


For most of human history the plight of the poor has been
easily excluded from the consciousness of those with the
power to act. Inaction was justified by elaborate theories that
the poor were by nature inferior or happy in their condition or
both. But human society confronts a new reality: pressures of
population and technology on the fragile balance of our
planet’s ecosystem are compelling the rich not only to re-
examine their own lifestyles, but also to take a greater interest
in the lifestyles of the poor whose conditions leave them
dependent for short-term survival on large families and the
destructive use of crop lands, forests, and watersheds—thus
posing a threat to the affluent more certain in its occurrence
and consequences than the threat of armed revolution. Poverty
has become salient to the powerful and the implications are far
reaching.

The text suggests that the poor affect the lives of the
affluent chiefly by doing what of the following?

A) They drain money from the government.


B) They damage the natural environment.
C) They make the wealthy feel guilty for their relative
affluence.
D) They threaten armed revolution.

© Jamboree Education Pvt. Ltd.


Compared to vibrant hybrid-tea colors,
antique rose colors tend to be muted. Their
stems are also shorter and buds are a bit
droopier. Their fragrance, however, is
unmatched. And unlike the hybrid-tea whose
long stems make for a rosebush that is rather
scraggly looking, antique rosebushes
_______________ handsomely landscaping
gardens.

Which choice completes the text with the


most logical transition?
A) can be grown in a variety of colors,
B) are lush and shapely,
C) can grow quite large,
D) tend to be less thorny,

© Jamboree Education Pvt. Ltd.


Compared to vibrant hybrid-tea colors,
antique rose colors tend to be muted. Their
stems are also shorter and buds are a bit
droopier. Their fragrance, however, is
unmatched. And unlike the hybrid-tea whose
long stems make for a rosebush that is rather
scraggly looking, antique rosebushes
_______________ handsomely landscaping
gardens.

Which choice completes the text with the


most logical transition?
A) can be grown in a variety of colors,
B) are lush and shapely,
C) can grow quite large,
D) tend to be less thorny,

© Jamboree Education Pvt. Ltd.


Efforts to understand how prehistoric animals
interacted with one another inevitably focus on
predatory relationships. Given this concentration, it
is not surprising that paleontologists – those who
study prehistoric life using fossil evidence – often
rely upon teeth to describe animals that lived in the
distant past and to determine their behaviors.
Through the evolutionary process, an animal's
teeth become precisely adapted for the acquisition
and initial processing of the specific types of foods
consumed by that animal. In addition, because
teeth are one of the hardest structures of the body,
they can remain in the environment for centuries
after an animal has died. Both of these qualities
make teeth quite _______ as prehistoric clues.

Which choice completes the text with the most


logical and precise word or phrase?

A) reliable
B) serviceable
C) precious
D) fixable

© Jamboree Education Pvt. Ltd.


Efforts to understand how prehistoric animals
interacted with one another inevitably focus on
predatory relationships. Given this concentration, it
is not surprising that paleontologists – those who
study prehistoric life using fossil evidence – often
rely upon teeth to describe animals that lived in the
distant past and to determine their behaviors.
Through the evolutionary process, an animal's
teeth become precisely adapted for the acquisition
and initial processing of the specific types of foods
consumed by that animal. In addition, because
teeth are one of the hardest structures of the body,
they can remain in the environment for centuries
after an animal has died. Both of these qualities
make teeth quite _______ as prehistoric clues.

Which choice completes the text with the most


logical and precise word or phrase?

A) reliable
B) serviceable
C) precious
D) fixable

© Jamboree Education Pvt. Ltd.


As I walk, alone and lonely, on the streets around my
small apartment, I feel a bond with virtually everyone
I see: the elderly man struggling to walk up the stoop
(a man who finds even the most basic tasks
challenging), the child playing with a soccer ball (a
_______ boy, experiencing easy joy), and even the
wrinkled woman stationed behind the cash register of
the convenience store, where she seems to be eight
days a week (a hard worker facing a daily struggle to
make a living). My own identity feels lost, wrapped
up in the lives of others. And yet I realize that as I
struggle to identify myself, I feel closer to the world
around me and less alone.

Which choice completes the text with the most


logical and precise word or phrase?

A) blithe
B) nervous
C) onerous
D) envious

© Jamboree Education Pvt. Ltd.


As I walk, alone and lonely, on the streets around my
small apartment, I feel a bond with virtually everyone
I see: the elderly man struggling to walk up the stoop
(a man who finds even the most basic tasks
challenging), the child playing with a soccer ball (a
_______ boy, experiencing easy joy), and even the
wrinkled woman stationed behind the cash register of
the convenience store, where she seems to be eight
days a week (a hard worker facing a daily struggle to
make a living). My own identity feels lost, wrapped
up in the lives of others. And yet I realize that as I
struggle to identify myself, I feel closer to the world
around me and less alone.

Which choice completes the text with the most


logical and precise word or phrase?

A) blithe
B) nervous
C) onerous
D) envious

© Jamboree Education Pvt. Ltd.


Text 1
People who work closely with animals readily acknowledge that How would the author of Text 2
they observe emotions in, or attribute emotions to, the animals they most likely respond to the
work with. Though this idea is derided by some in the scientific underlined portion of Text 1?
community, it is important to note that the most successful animal
A) By arguing that animals actually
trainers are the ones who understand the "mood" of an animal –
feel no emotion.
who are comfortable reading the emotional state of their charges. A
B) By questioning the objectivity of
horse trainer had better know whether a horse will take coaxing or
trainers who claim to know their
commanding on a given day, and any trainer will tell you that
charges' emotions.
different approaches are effective with different individuals, even
C) By arguing that little is actually
those from the same breed. These trainers attribute the difference
known about animals' emotional
to the animals' emotional tendencies.
relationships with one another.
Text 2 D) By suggesting other possible
explanations for an animal
The Oxford Companion to Animal Behavior advises animal
acting differently on different
behaviorists: "One is well advised to study the behavior [of an
days.
animal], rather than attempting to get at any underlying emotion."
Whether or not animals experience emotions similar to those that
humans do, the fact is that there is no scientific way to quantify
and categorize them. If animals do experience emotion, it is certain
that they do not express it in the same way we do. Accordingly,
making any conclusions about emotions based on behavior
amounts to nothing more than guesswork and wishful
anthropomorphizing.

© Jamboree Education Pvt. Ltd.


Text 1
People who work closely with animals readily acknowledge that How would the author of Text 2
they observe emotions in, or attribute emotions to, the animals they most likely respond to the
work with. Though this idea is derided by some in the scientific underlined portion of Text 1?
community, it is important to note that the most successful animal
A) By arguing that animals actually
trainers are the ones who understand the "mood" of an animal –
feel no emotion.
who are comfortable reading the emotional state of their charges. A
B) By questioning the objectivity of
horse trainer had better know whether a horse will take coaxing or
trainers who claim to know their
commanding on a given day, and any trainer will tell you that
charges' emotions.
different approaches are effective with different individuals, even
C) By arguing that little is actually
those from the same breed. These trainers attribute the difference
known about animals' emotional
to the animals' emotional tendencies.
relationships with one another.
Text 2 D) By suggesting other possible
explanations for an animal
The Oxford Companion to Animal Behavior advises animal
acting differently on different
behaviorists: "One is well advised to study the behavior [of an
days.
animal], rather than attempting to get at any underlying emotion."
Whether or not animals experience emotions similar to those that
humans do, the fact is that there is no scientific way to quantify
and categorize them. If animals do experience emotion, it is certain
that they do not express it in the same way we do. Accordingly,
making any conclusions about emotions based on behavior
amounts to nothing more than guesswork and wishful
anthropomorphizing.

© Jamboree Education Pvt. Ltd.


A mural titled Celadon Remnants is artist Jean
Shin’s homage to the Korean American community
in Flushing. When she was commissioned by the
Metropolitan Transportation Authority of New
York City to produce a site-specific artwork,
___________________ as an American and a
Korean. She chose to use traditional celadon
pottery, albeit in a new way.

Which choice completes the text so that it


conforms to the conventions of Standard
English?
A) visually representing her dual identity was a
means sought by Shin
B) Shin sought a means of visually representing
her dual identity
C) sought by Shin was a means of visually
representing her dual identity
D) a means of visually representing her dual
identity was sought by Shin

© Jamboree Education Pvt. Ltd.


A mural titled Celadon Remnants is artist Jean
Shin’s homage to the Korean American community
in Flushing. When she was commissioned by the
Metropolitan Transportation Authority of New
York City to produce a site-specific artwork,
___________________ as an American and a
Korean. She chose to use traditional celadon
pottery, albeit in a new way.

Which choice completes the text so that it


conforms to the conventions of Standard
English?
A) visually representing her dual identity was a
means sought by Shin
B) Shin sought a means of visually representing
her dual identity
C) sought by Shin was a means of visually
representing her dual identity
D) a means of visually representing her dual
identity was sought by Shin

© Jamboree Education Pvt. Ltd.


Using computational modeling,
MacIver and Schmitz simulated the
visibility of a 10-centimeter black
Disc through water and through air.
When the view of the disc through
water was simulated, a tripling of
pupil size from 5 to 15 millimeters
yielded an increase in visual range of
__________________________

Which choice most effectively uses


data from the graph to complete the
text?
A) between 4 and 5 meters.
B) between 200 and 300 meters.
C) more than 400 meters.
D) less than 0.5 meter.

© Jamboree Education Pvt. Ltd.


Using computational modeling,
MacIver and Schmitz simulated the
visibility of a 10-centimeter black
Disc through water and through air.
When the view of the disc through
water was simulated, a tripling of
pupil size from 5 to 15 millimeters
yielded an increase in visual range of
__________________________

Which choice most effectively uses


data from the graph to complete the
text?
A) between 4 and 5 meters.
B) between 200 and 300 meters.
C) more than 400 meters.
D) less than 0.5 meter.

© Jamboree Education Pvt. Ltd.


Two decades after the Emerald River Dam was built, none of
the eight fish species native to the Emerald River was still
reproducing adequately in the river below the dam. Since the
dam reduced the annual range of water temperature in the river
below the dam from 50 degrees to 6 degrees, scientists have
hypothesized that this drop in water temperatures must be
preventing the native species from beginning the reproductive
cycle.

Which of the following statements, if true, would support the


scientists’ hypothesis?

A) The native fish species were still able to reproduce only in


side streams of the river below the dam where the annual
temperature range remains approximately 50 degrees.
B) Before the dam was built, the Emerald River annually
overflowed its banks, creating backwaters that were critical
breeding areas for the native species of fish.
C) The lowest recorded temperature of the Emerald River
before the dam was built was 34 degrees, whereas the
lowest recorded temperature of the river after the dam was
built has been 43 degrees.
D) Five of the fish species native to the Emerald River are not
native to any other river in North America.

© Jamboree Education Pvt. Ltd.


Two decades after the Emerald River Dam was built, none of
the eight fish species native to the Emerald River was still
reproducing adequately in the river below the dam. Since the
dam reduced the annual range of water temperature in the river
below the dam from 50 degrees to 6 degrees, scientists have
hypothesized that this drop in water temperatures must be
preventing the native species from beginning the reproductive
cycle.

Which of the following statements, if true, would support the


scientists’ hypothesis?

A) The native fish species were still able to reproduce only in


side streams of the river below the dam where the annual
temperature range remains approximately 50 degrees.
B) Before the dam was built, the Emerald River annually
overflowed its banks, creating backwaters that were critical
breeding areas for the native species of fish.
C) The lowest recorded temperature of the Emerald River
before the dam was built was 34 degrees, whereas the
lowest recorded temperature of the river after the dam was
built has been 43 degrees.
D) Five of the fish species native to the Emerald River are not
native to any other river in North America.

© Jamboree Education Pvt. Ltd.


In a culture in which email, cell phones, pagers, and Internet
chat rooms have become everyday modes of
communication, we are on the ____________ of breaking
down all barriers to the complete and constant transfer of
information. But if we seem to be moving toward unfettered
union, we could also be seen as being more isolated as
individuals than we have ever been before. As we entered
what has come to be known as the "Information Age," every
new technological advance was touted as the solution to a
myriad of problems. With more ways to pass along all types
of information, the communications industry reasons, we
will become more efficient at work, more in touch with
distant family, and more likely to meet new people. With
such expanded access to the world at large, we can only
become more adept social animals.

Which choice completes the text with the most logical and
precise word or phrase?
A) joint
B) pinnacle
C) togetherness
D) brink
© Jamboree Education Pvt. Ltd.
In a culture in which email, cell phones, pagers, and Internet
chat rooms have become everyday modes of
communication, we are on the ____________ of breaking
down all barriers to the complete and constant transfer of
information. But if we seem to be moving toward unfettered
union, we could also be seen as being more isolated as
individuals than we have ever been before. As we entered
what has come to be known as the "Information Age," every
new technological advance was touted as the solution to a
myriad of problems. With more ways to pass along all types
of information, the communications industry reasons, we
will become more efficient at work, more in touch with
distant family, and more likely to meet new people. With
such expanded access to the world at large, we can only
become more adept social animals.

Which choice completes the text with the most logical and
precise word or phrase?
A) joint
B) pinnacle
C) togetherness
D) brink
© Jamboree Education Pvt. Ltd.
Lithography is the art of drawing with a greasy
substance, usually crayon, on a stone, metal, or paper
surface, and then printing. It is based on the fact that
grease attracts grease and is repelled by water. It is
the most direct of all the graphic arts, for in practicing
it the artist first sees the exact value of each line that
he draws and then has his drawing reproduced so
accurately that it may truly be said to have been
multiplied. In making either an etching, a process in
which a drawing is engraved on a metal plate through
a thin film of wax, or a woodblock, in which the
drawing is carved in wood, the artist must wait for a
print to estimate his work fairly. When a lithograph is
made, the artist’s drawing grows in definite values
under his eyes and he can make changes in it as he
works.

Which of the following does the text state is the


reason many artists like to use lithography to
reproduce their drawings?
A) Because they know in advance the value of each
picture.
B) Because they often get unexpected results.
C) Because they can get clearer enlargements.
D) Because they can make alterations and
corrections.
© Jamboree Education Pvt. Ltd.
Lithography is the art of drawing with a greasy
substance, usually crayon, on a stone, metal, or paper
surface, and then printing. It is based on the fact that
grease attracts grease and is repelled by water. It is
the most direct of all the graphic arts, for in practicing
it the artist first sees the exact value of each line that
he draws and then has his drawing reproduced so
accurately that it may truly be said to have been
multiplied. In making either an etching, a process in
which a drawing is engraved on a metal plate through
a thin film of wax, or a woodblock, in which the
drawing is carved in wood, the artist must wait for a
print to estimate his work fairly. When a lithograph is
made, the artist’s drawing grows in definite values
under his eyes and he can make changes in it as he
works.

Which of the following does the text state is the


reason many artists like to use lithography to
reproduce their drawings?
A) Because they know in advance the value of each
picture.
B) Because they often get unexpected results.
C) Because they can get clearer enlargements.
D) Because they can make alterations and
corrections.
© Jamboree Education Pvt. Ltd.
Social history sometimes suffers from the reproach With which of the following contrasts
that it is vague and general, unable to compete with does the text primarily deal with?
the attractions of political history either for the
student or for the general reader, because of its lack A) Between accounts of modern times
of outstanding personalities. In point of fact there is and accounts of ancient times
often as much material for reconstructing the life of B) Between fictionalized historical
some quite ordinary person as there is for writing the accounts and objective historical
history of Robert of Normandy or Philippa of
analyses
Hainault; and the lives of ordinary people so
C) Between the concerns of the affluent
reconstructed are, if less spectacular, certainly not
less interesting. I believe that social history lends and the concerns of the poor
itself particularly to what may be called a personal D) Between depictions of ordinary
treatment, and that the past may be made to live citizens and depictions of political
again for the general reader more effectively by leaders and institutions
personifying it than by presenting it in the form of
learned treatises on the development of the manor or
on medieval trade, essential as these are to the
specialist. For history, after all, is valuable only in so
far as it lives, and Maeterlinck’s cry, “There are no
dead,” should always be the historian’s motto.

© Jamboree Education Pvt. Ltd.


Social history sometimes suffers from the reproach With which of the following contrasts
that it is vague and general, unable to compete with does the text primarily deal with?
the attractions of political history either for the
student or for the general reader, because of its lack A) Between accounts of modern times
of outstanding personalities. In point of fact there is and accounts of ancient times
often as much material for reconstructing the life of B) Between fictionalized historical
some quite ordinary person as there is for writing the accounts and objective historical
history of Robert of Normandy or Philippa of
analyses
Hainault; and the lives of ordinary people so
C) Between the concerns of the affluent
reconstructed are, if less spectacular, certainly not
less interesting. I believe that social history lends and the concerns of the poor
itself particularly to what may be called a personal D) Between depictions of ordinary
treatment, and that the past may be made to live citizens and depictions of political
again for the general reader more effectively by leaders and institutions
personifying it than by presenting it in the form of
learned treatises on the development of the manor or
on medieval trade, essential as these are to the
specialist. For history, after all, is valuable only in so
far as it lives, and Maeterlinck’s cry, “There are no
dead,” should always be the historian’s motto.

© Jamboree Education Pvt. Ltd.


In order to be “seen,” an object or event must
emit or reflect light particles, or photons, that
travel relatively unimpeded to a detector, such
as a telescope or your retina. You see a candle
flame because its excited atoms produce
photons that zip into your eyeballs, and you see
a painting because myriad photons from a light
source reflect off of it. Sometimes, however,
photons can be absorbed by atoms or scattered
by electrons, thwarting their arrival at a detector
and rendering their source invisible.

Which choice best states the main purpose of


the text?
A) It describes a biological process.
B) It introduces experimental evidences.
C) It explains a physical phenomenon.
D) It refutes a misconception.

© Jamboree Education Pvt. Ltd.


In order to be “seen,” an object or event must
emit or reflect light particles, or photons, that
travel relatively unimpeded to a detector, such
as a telescope or your retina. You see a candle
flame because its excited atoms produce
photons that zip into your eyeballs, and you see
a painting because myriad photons from a light
source reflect off of it. Sometimes, however,
photons can be absorbed by atoms or scattered
by electrons, thwarting their arrival at a detector
and rendering their source invisible.

Which choice best states the main purpose of


the text?
A) It describes a biological process.
B) It introduces experimental evidences.
C) It explains a physical phenomenon.
D) It refutes a misconception.

© Jamboree Education Pvt. Ltd.


Critics of new media sometimes use science itself to
press their case, citing research that shows how
“experience can change the brain.” But cognitive
neuroscientists roll their eyes at such talk. Yes, every
time we learn a fact or skill the wiring of the brain
changes; it’s not as if the information is stored in the
pancreas. But the existence of neural plasticity does
not mean the brain is a blob of clay pounded into
shape by experience. Experience does not revamp the
basic information-processing capacities of the brain.
Speed-reading programs have long claimed to do just
that, but the verdict was rendered by Woody Allen
after he read Leo Tolstoy’s famously long novel War
and Peace in one sitting: “It was about Russia.”

Which choice best states the central claim of the


text?
A) Neural plasticity is a myth promoted by the
critics of new media.
B) New experiences do change how the brain
functions.
C) Basic cognitive capacities of brain do not change
with new experiences.
D) Woody Allen’s reading of Leo Tolstoy’s novel
War and Peace was ineffective.
© Jamboree Education Pvt. Ltd.
Critics of new media sometimes use science itself to
press their case, citing research that shows how
“experience can change the brain.” But cognitive
neuroscientists roll their eyes at such talk. Yes, every
time we learn a fact or skill the wiring of the brain
changes; it’s not as if the information is stored in the
pancreas. But the existence of neural plasticity does
not mean the brain is a blob of clay pounded into
shape by experience. Experience does not revamp the
basic information-processing capacities of the brain.
Speed-reading programs have long claimed to do just
that, but the verdict was rendered by Woody Allen
after he read Leo Tolstoy’s famously long novel War
and Peace in one sitting: “It was about Russia.”

Which choice best states the central claim of the


text?
A) Neural plasticity is a myth promoted by the
critics of new media.
B) New experiences do change how the brain
functions.
C) Basic cognitive capacities of brain do not change
with new experiences.
D) Woody Allen’s reading of Leo Tolstoy’s novel
War and Peace was ineffective.
© Jamboree Education Pvt. Ltd.
A century ago, opponents of women's suffrage in the
United States scoffed at the notion that extending the
vote to women would make any difference. "Women
will vote with their husbands" was the commonly
accepted wisdom. This was an argument made in the
absence of evidence, as women did not yet have the
vote. Ever since women won the vote, researchers
have been keeping close track of female voting
behavior. A "gender gap" in voting behavior has been
found in the United States as in many other countries.
In the United States, the 1994 and 1996 elections
showed the largest gaps ever between candidates
favored by women and those favored by men.

Which choice best states the primary purpose of the


text?
A) It presents a concise history of the women's
suffrage movement.
B) It shows the inaccuracy of a prediction about
women's suffrage.
C) It discusses the resistance women faced in
acquiring the vote.
D) It explains the differences in voting behavior
between men and women.

© Jamboree Education Pvt. Ltd.


A century ago, opponents of women's suffrage in the
United States scoffed at the notion that extending the
vote to women would make any difference. "Women
will vote with their husbands" was the commonly
accepted wisdom. This was an argument made in the
absence of evidence, as women did not yet have the
vote. Ever since women won the vote, researchers
have been keeping close track of female voting
behavior. A "gender gap" in voting behavior has been
found in the United States as in many other countries.
In the United States, the 1994 and 1996 elections
showed the largest gaps ever between candidates
favored by women and those favored by men.

Which choice best states the primary purpose of the


text?
A) It presents a concise history of the women's
suffrage movement.
B) It shows the inaccuracy of a prediction about
women's suffrage.
C) It discusses the resistance women faced in
acquiring the vote.
D) It explains the differences in voting behavior
between men and women.

© Jamboree Education Pvt. Ltd.


Text 1
If the most dire of the widespread global warming theories were Which choice best states the
accurate, the polar ice caps would currently be receding central claim of both the
significantly. Recent studies, however, have demonstrated that the texts?
Arctic ice shelf is not only maintaining its mass, but also growing. A) Global warming needs to
In addition, satellite temperature readings, considered by many to be better controlled.
be more reliable than surface temperatures taken by humans under B) The evidence supporting
varying conditions, indicate no global warming of the lower
global warming theories is
atmosphere. Environmental organizations have reported that Arctic
Sea ice declined by 14 percent from 1978 to 1998. Yet a careful questionable.
review of this research reveals that almost this entire drop occurred C) It is important to study a
during a period of only one year, suggesting this temperature period of at least 20 years.
change was a result of an anomaly rather than a growing trend. D) The polar ice caps are
Text 2 affected by global
temperature changes.
Following on the heels of convincing new evidence that the Arctic
ice cap is rapidly melting-some studies, using satellite temperature
readings, mark the decline at 20 percent over the past 20 years-
efforts have begun to pass legislation that would reduce global
warming pollution and curb climate change. Some of the proposed
bills would require the manufacturing sector to restrict carbon
dioxide emissions that, over the years, have contributed to a layer
of pollution scientists believe traps a substantial portion of the sun's
heat. It is significant that such discussions are taking place in the
United States, the estimated producer of 25 percent of the world's
carbon dioxide pollution.

© Jamboree Education Pvt. Ltd.


Text 1
If the most dire of the widespread global warming theories were Which choice best states the
accurate, the polar ice caps would currently be receding central claim of both the
significantly. Recent studies, however, have demonstrated that the texts?
Arctic ice shelf is not only maintaining its mass, but also growing. A) Global warming needs to
In addition, satellite temperature readings, considered by many to be better controlled.
be more reliable than surface temperatures taken by humans under B) The evidence supporting
varying conditions, indicate no global warming of the lower
global warming theories is
atmosphere. Environmental organizations have reported that Arctic
Sea ice declined by 14 percent from 1978 to 1998. Yet a careful questionable.
review of this research reveals that almost this entire drop occurred C) It is important to study a
during a period of only one year, suggesting this temperature period of at least 20 years.
change was a result of an anomaly rather than a growing trend. D) The polar ice caps are
Text 2 affected by global
temperature changes.
Following on the heels of convincing new evidence that the Arctic
ice cap is rapidly melting-some studies, using satellite temperature
readings, mark the decline at 20 percent over the past 20 years-
efforts have begun to pass legislation that would reduce global
warming pollution and curb climate change. Some of the proposed
bills would require the manufacturing sector to restrict carbon
dioxide emissions that, over the years, have contributed to a layer
of pollution scientists believe traps a substantial portion of the sun's
heat. It is significant that such discussions are taking place in the
United States, the estimated producer of 25 percent of the world's
carbon dioxide pollution.

© Jamboree Education Pvt. Ltd.


Male bowerbirds construct elaborately decorated
nests, or bowers. Basing their judgment on the fact
that different local populations of bowerbirds of the
same species build bowers that exhibit different
building and decorative styles, researchers have
concluded that the bowerbirds' building styles are a
culturally acquired, rather than a genetically
transmitted, trait.

Which of the following, if true, would strengthen


the conclusion drawn by the researchers?
A) There are more common characteristics than
there are differences among the bowerbuilding
styles of the local bowerbird population that has
been studied most extensively.
B) Young male bowerbirds are inept at
bowerbuilding and apparently spend years
watching their elders before becoming
accomplished in the local bower style.
C) The bowers of one species of bowerbird lack the
towers and ornamentation characteristic of the
bowers of most other species of bowerbird.
D) Bowerbirds are found only in New Guinea and
Australia, where local populations of the birds
apparently seldom have contact with one another.

© Jamboree Education Pvt. Ltd.


Male bowerbirds construct elaborately decorated
nests, or bowers. Basing their judgment on the fact
that different local populations of bowerbirds of the
same species build bowers that exhibit different
building and decorative styles, researchers have
concluded that the bowerbirds' building styles are a
culturally acquired, rather than a genetically
transmitted, trait.

Which of the following, if true, would strengthen


the conclusion drawn by the researchers?
A) There are more common characteristics than
there are differences among the bowerbuilding
styles of the local bowerbird population that has
been studied most extensively.
B) Young male bowerbirds are inept at
bowerbuilding and apparently spend years
watching their elders before becoming
accomplished in the local bower style.
C) The bowers of one species of bowerbird lack the
towers and ornamentation characteristic of the
bowers of most other species of bowerbird.
D) Bowerbirds are found only in New Guinea and
Australia, where local populations of the birds
apparently seldom have contact with one another.

© Jamboree Education Pvt. Ltd.


Many animals produce sounds that resemble
human music. In the 1960s, Cornell researchers
Katy and Roger Payne found that humpback
whales create songs consisting of long, intricate
__________ researchers have observed
chimpanzees producing signature rhythms with
their hands or feet.

Which choice completes the text so that it


conforms to the conventions of Standard
English?
A) sequences similarly
B) sequences, similarly,
C) sequences; similarly,
D) sequences similarly,

© Jamboree Education Pvt. Ltd.


Many animals produce sounds that resemble
human music. In the 1960s, Cornell researchers
Katy and Roger Payne found that humpback
whales create songs consisting of long, intricate
__________ researchers have observed
chimpanzees producing signature rhythms with
their hands or feet.

Which choice completes the text so that it


conforms to the conventions of Standard
English?
A) sequences similarly
B) sequences, similarly,
C) sequences; similarly,
D) sequences similarly,

© Jamboree Education Pvt. Ltd.


Like its biological counterpart, AQUA2 has
flippers that allow it to glide through the water,
dive to the ocean floor, and ascend from the
bottom. Unlike thruster-powered robots,
_______________ altering the positions of its
flippers.

Which choice completes the text so that it


conforms to the conventions of Standard
English?
A) subtle changes in course can be made by
AQUA2 simply
B) a possibility exists for subtle changes in
course to be made simply by AQUA2
C) AQUA2 can make subtle changes in course
simply by
D) subtle changes in course can be made
simply by AQUA2

© Jamboree Education Pvt. Ltd.


Like its biological counterpart, AQUA2 has
flippers that allow it to glide through the water,
dive to the ocean floor, and ascend from the
bottom. Unlike thruster-powered robots,
_______________ altering the positions of its
flippers.

Which choice completes the text so that it


conforms to the conventions of Standard
English?
A) subtle changes in course can be made by
AQUA2 simply
B) a possibility exists for subtle changes in
course to be made simply by AQUA2
C) AQUA2 can make subtle changes in course
simply by
D) subtle changes in course can be made
simply by AQUA2

© Jamboree Education Pvt. Ltd.


Few figures in this century have been so admired and
so savagely attacked. Ayn Rand is viewed as goddess
and malefactor, as a seminal genius and an ominously
dangerous corrupter of the young, as the mightiest of
voices for reason and the destroyer of traditional
values, as the espouser of joy and the exponent of
mindless greed, as the great defender of freedom and
the introducer of malevolent values into the
mainstream of American thought. It is all but
impossible to find a neutral voice among the millions
who have read her works; each reader takes an
unequivocal stand for or against what she represents.
When her name is mentioned in any gathering, it is
met with explosions of grateful, loving admiration or
enraged disapproval.

The text suggests which of the following about


those who have read Ayn Rand’s works?

A) They are united in their admiration of her ideals.


B) They are polarized.
C) They are critical of her literary style.
D) They are respectful of her life experiences.

© Jamboree Education Pvt. Ltd.


Few figures in this century have been so admired and
so savagely attacked. Ayn Rand is viewed as goddess
and malefactor, as a seminal genius and an ominously
dangerous corrupter of the young, as the mightiest of
voices for reason and the destroyer of traditional
values, as the espouser of joy and the exponent of
mindless greed, as the great defender of freedom and
the introducer of malevolent values into the
mainstream of American thought. It is all but
impossible to find a neutral voice among the millions
who have read her works; each reader takes an
unequivocal stand for or against what she represents.
When her name is mentioned in any gathering, it is
met with explosions of grateful, loving admiration or
enraged disapproval.

The text suggests which of the following about


those who have read Ayn Rand’s works?

A) They are united in their admiration of her ideals.


B) They are polarized.
C) They are critical of her literary style.
D) They are respectful of her life experiences.

© Jamboree Education Pvt. Ltd.


As hominid history has unfolded, social and cultural means of
adaptation have become increasingly important. In this
process, humans have devised diverse ways of coping with the
range of environments and social systems they have occupied
in time and space. The rate of cultural change has accelerated,
particularly during the past 10,000 years. For millions of years,
hunting and gathering of nature’s bounty – foraging – was the
sole basis of hominid subsistence. However, it took only a few
thousand years for food production (cultivation of plants and
domestication of animals), which originated in the Middle East
10,000 to 12,000 years ago, to replace foraging in most areas.
People started producing their own food, planting crops and
stockbreeding animals, rather than simply taking what nature
had to offer.

The text indicates which of the following about the practice


of food production among hominids?

A) It was not as widely available as the practice of food


gathering.
B) It was introduced over one million years ago.
C) It developed more rapidly than the practice of food
gathering.
D) It damaged the environment in which the hominids lived.

© Jamboree Education Pvt. Ltd.


As hominid history has unfolded, social and cultural means of
adaptation have become increasingly important. In this
process, humans have devised diverse ways of coping with the
range of environments and social systems they have occupied
in time and space. The rate of cultural change has accelerated,
particularly during the past 10,000 years. For millions of years,
hunting and gathering of nature’s bounty – foraging – was the
sole basis of hominid subsistence. However, it took only a few
thousand years for food production (cultivation of plants and
domestication of animals), which originated in the Middle East
10,000 to 12,000 years ago, to replace foraging in most areas.
People started producing their own food, planting crops and
stockbreeding animals, rather than simply taking what nature
had to offer.

The text indicates which of the following about the practice


of food production among hominids?

A) It was not as widely available as the practice of food


gathering.
B) It was introduced over one million years ago.
C) It developed more rapidly than the practice of food
gathering.
D) It damaged the environment in which the hominids lived.

© Jamboree Education Pvt. Ltd.


The contributions of Charles Young
_____________ counted among the most
significant in the National Park’s history. In
2003, the National Park Service decided to
formally recognize the efforts of Captain Young
(who had been promoted to lieutenant colonel
in 1916). Today, those who visit the park use
Young’s road to reach the Giant Forest, where,
among the trees dedicated to and named for US
generals and presidents, the Colonel Young
Tree also stands.

Which choice completes the text so that it


conforms to the conventions of Standard
English?
A) has been
B) was
C) are
D) is

© Jamboree Education Pvt. Ltd.


The contributions of Charles Young
_____________ counted among the most
significant in the National Park’s history. In
2003, the National Park Service decided to
formally recognize the efforts of Captain Young
(who had been promoted to lieutenant colonel
in 1916). Today, those who visit the park use
Young’s road to reach the Giant Forest, where,
among the trees dedicated to and named for US
generals and presidents, the Colonel Young
Tree also stands.

Which choice completes the text so that it


conforms to the conventions of Standard
English?
A) has been
B) was
C) are
D) is

© Jamboree Education Pvt. Ltd.


My daughter, Olivia, and I were going to
college. Not together at the same school, thank
goodness, just at the same time, but she didn't
exactly know about my plans yet. There were a
few things that needed work in this
arrangement. Any mother who has an eighteen-
year-old daughter would completely understand
why I didn't mention my decision to go back to
college to Olivia. What? I can't believe it. Are
you actually copying me? Don't you think you
should consider getting your own life? It wasn't
that I planned never to tell her. I just figured I'd
wait a bit – until we’d had a little time to miss
each other.

Why does the narrator use the questions in the


text?
A) To voice some pressing concerns.
B) To begin a discussion.
C) To admit to some personal qualms.
D) To characterize a likely response.

© Jamboree Education Pvt. Ltd.


My daughter, Olivia, and I were going to
college. Not together at the same school, thank
goodness, just at the same time, but she didn't
exactly know about my plans yet. There were a
few things that needed work in this
arrangement. Any mother who has an eighteen-
year-old daughter would completely understand
why I didn't mention my decision to go back to
college to Olivia. What? I can't believe it. Are
you actually copying me? Don't you think you
should consider getting your own life? It wasn't
that I planned never to tell her. I just figured I'd
wait a bit – until we’d had a little time to miss
each other.

Why does the narrator use the questions in the


text?
A) To voice some pressing concerns.
B) To begin a discussion.
C) To admit to some personal qualms.
D) To characterize a likely response.

© Jamboree Education Pvt. Ltd.


In China, calligraphy – the ancient practice
of artistically writing Chinese characters –
has long been revered as a supreme form of
visual art. Chinese calligraphy also
____________ the more practical purpose of
communicating language in a variety of
scripts.

Which choice completes the text so that it


conforms to the conventions of Standard
English?
A) serves
B) serve
C) are serving
D) have served

© Jamboree Education Pvt. Ltd.


In China, calligraphy – the ancient practice
of artistically writing Chinese characters –
has long been revered as a supreme form of
visual art. Chinese calligraphy also
____________ the more practical purpose of
communicating language in a variety of
scripts.

Which choice completes the text so that it


conforms to the conventions of Standard
English?
A) serves
B) serve
C) are serving
D) have served

© Jamboree Education Pvt. Ltd.


My brother forced me to memorize four According to the text, which of the
Chinese classics – The Analects of Confucius, following is one significant difference
The Great Learning, The Doctrine of the between the narrator’s experience with
Mean, and The Works of Mencius – none of
The Count of Monte Cristo and The
which I understood. My reading was not
confined exclusively to Chinese literature, Great Learning?
however. My pre-teen tastes tended toward A) The author learnt about Chinese
romantic European novels, many of them
nobility in one and about Chinese
translated into Chinese. I preferred to read the
peasants in the other.
English translations of books such as The
Three Musketeers and The Count of Monte B) The author studied one book
Cristo. It was this latter title that made the willingly and the other unwillingly.
deepest impression on me. I knew whole C) The author condemned violence as a
passages by heart, and even took the trouble to virtue in one and espoused it in the
reread the book in Chinese, the title of which other.
translates as The Vengeance and the Gratitude
D) The author studied the former in one
of the Count of Monte Cristo, reflecting a
more Chinese concept in its translation. In language and the latter in two
either version, I loved this strong and willful languages.
character that was so undeserving of all the
misfortune that befell him.

© Jamboree Education Pvt. Ltd.


My brother forced me to memorize four According to the text, which of the
Chinese classics – The Analects of Confucius, following is one significant difference
The Great Learning, The Doctrine of the between the narrator’s experience with
Mean, and The Works of Mencius – none of
The Count of Monte Cristo and The
which I understood. My reading was not
confined exclusively to Chinese literature, Great Learning?
however. My pre-teen tastes tended toward A) The author learnt about Chinese
romantic European novels, many of them
nobility in one and about Chinese
translated into Chinese. I preferred to read the
peasants in the other.
English translations of books such as The
Three Musketeers and The Count of Monte B) The author studied one book
Cristo. It was this latter title that made the willingly and the other unwillingly.
deepest impression on me. I knew whole C) The author condemned violence as a
passages by heart, and even took the trouble to virtue in one and espoused it in the
reread the book in Chinese, the title of which other.
translates as The Vengeance and the Gratitude
D) The author studied the former in one
of the Count of Monte Cristo, reflecting a
more Chinese concept in its translation. In language and the latter in two
either version, I loved this strong and willful languages.
character that was so undeserving of all the
misfortune that befell him.

© Jamboree Education Pvt. Ltd.


Many breakfast cereals are fortified with vitamin
supplements. Some of these cereals provide 100
percent of the recommended daily requirement of
vitamins. Despite this, most nutritionists claim that a
well-balanced breakfast, including a variety of
foods, is a better source of those vitamins than are
such fortified breakfast cereals alone.

Which of the following, if true, would strongly


support the nutritionists’ claim?
A) People who regularly eat cereals fortified with
vitamin supplements sometimes neglect to eat
the foods in which the vitamins occur naturally.
B) Foods often must be fortified with vitamin
supplements because naturally occurring
vitamins are removed during processing.
C) In many foods, the natural combination of
vitamins with other nutrients makes those
vitamins more usable by the body than are
vitamins added in vitamin supplements.
D) Cereals containing vitamin supplements are no
harder to digest than similar cereals without
added vitamins.
© Jamboree Education Pvt. Ltd.
Many breakfast cereals are fortified with vitamin
supplements. Some of these cereals provide 100
percent of the recommended daily requirement of
vitamins. Despite this, most nutritionists claim that a
well-balanced breakfast, including a variety of
foods, is a better source of those vitamins than are
such fortified breakfast cereals alone.

Which of the following, if true, would strongly


support the nutritionists’ claim?
A) People who regularly eat cereals fortified with
vitamin supplements sometimes neglect to eat
the foods in which the vitamins occur naturally.
B) Foods often must be fortified with vitamin
supplements because naturally occurring
vitamins are removed during processing.
C) In many foods, the natural combination of
vitamins with other nutrients makes those
vitamins more usable by the body than are
vitamins added in vitamin supplements.
D) Cereals containing vitamin supplements are no
harder to digest than similar cereals without
added vitamins.
© Jamboree Education Pvt. Ltd.
Professional reviewers’ commitment to
fairness and accuracy should be appealing
to consumers since, according to market
research, people have many reservations
about online customer reviews. For
example, a 2014 study found that 50
percent of those surveyed believed that
businesses sometimes write negative
reviews about competitors, and a further 18
percent believed businesses did so often.
The individuals surveyed also thought that
businesses write positive reviews of their
own products or services, with 43 percent
of respondents saying this happens
sometimes and _______________________

Which choice most effectively uses data


from the graph to complete the text?
A) 21 percent saying it happens often.
B) 36 percent saying it happens often.
C) 9 percent saying it happens rarely.
D) 11 percent saying it never happens.

© Jamboree Education Pvt. Ltd.


Professional reviewers’ commitment to
fairness and accuracy should be appealing
to consumers since, according to market
research, people have many reservations
about online customer reviews. For
example, a 2014 study found that 50
percent of those surveyed believed that
businesses sometimes write negative
reviews about competitors, and a further 18
percent believed businesses did so often.
The individuals surveyed also thought that
businesses write positive reviews of their
own products or services, with 43 percent
of respondents saying this happens
sometimes and _______________________

Which choice most effectively uses data


from the graph to complete the text?
A) 21 percent saying it happens often.
B) 36 percent saying it happens often.
C) 9 percent saying it happens rarely.
D) 11 percent saying it never happens.

© Jamboree Education Pvt. Ltd.


____________ in Biscayne Bay. His father had
purchased the land for $300 in 1897, and Jones
grew up there, cultivating pineapples and Key
limes. In 1935, Jones began guiding fishing
trips; his knowledge of fishing earned him the
reputation for being the area’s best fishing
guide. Among his clientele were several US
presidents, including Hoover, Kennedy, and
Nixon.

Which choice completes the text so that it


conforms to the conventions of Standard
English?
A) Porgy Key had always been home for Jones,
B) Jones had lived on Porgy Key his entire life,
C) Porgy Key always was a home for Jones,
D) Jones had always lived on Porgy Key,

© Jamboree Education Pvt. Ltd.


____________ in Biscayne Bay. His father had
purchased the land for $300 in 1897, and Jones
grew up there, cultivating pineapples and Key
limes. In 1935, Jones began guiding fishing
trips; his knowledge of fishing earned him the
reputation for being the area’s best fishing
guide. Among his clientele were several US
presidents, including Hoover, Kennedy, and
Nixon.

Which choice completes the text so that it


conforms to the conventions of Standard
English?
A) Porgy Key had always been home for Jones,
B) Jones had lived on Porgy Key his entire life,
C) Porgy Key always was a home for Jones,
D) Jones had always lived on Porgy Key,

© Jamboree Education Pvt. Ltd.


Traditional codes of medical ethics, as well According to the text, on which of the
other traditional expositions, emphasize the following are the lay attitudes
moral obligations of health-care professionals becoming increasingly focused?
rather than the moral rights of patients.
Physicians are expected to perform those A) The behavior of professionals.
actions which will benefit their patients and to B) Emotional health.
refrain from performing those that will harm C) The rights of physicians.
them. Thus, traditional medical ethics stresses
D) The rights of those receiving
two fundamental principles as governing the
treatment.
physician-patient relationship—the principles
of beneficence and nonmaleficence. Both of
these are expressed in the dictum, “Benefit and
do no harm to the patient.” In contrast, recent
discussions of medical ethics speak more and
more about the moral rights of patients,
especially about the right to make their own
medical decisions. These discussions emphasize
the right of patients to act as autonomous
decision makers, determining for themselves
what will be done to their bodies. This change
of emphasis reflects a growing change in lay
attitudes toward health-care professionals,
especially physicians.
© Jamboree Education Pvt. Ltd.
Traditional codes of medical ethics, as well According to the text, on which of the
other traditional expositions, emphasize the following are the lay attitudes
moral obligations of health-care professionals becoming increasingly focused?
rather than the moral rights of patients.
Physicians are expected to perform those A) The behavior of professionals.
actions which will benefit their patients and to B) Emotional health.
refrain from performing those that will harm C) The rights of physicians.
them. Thus, traditional medical ethics stresses
D) The rights of those receiving
two fundamental principles as governing the
treatment.
physician-patient relationship—the principles
of beneficence and nonmaleficence. Both of
these are expressed in the dictum, “Benefit and
do no harm to the patient.” In contrast, recent
discussions of medical ethics speak more and
more about the moral rights of patients,
especially about the right to make their own
medical decisions. These discussions emphasize
the right of patients to act as autonomous
decision makers, determining for themselves
what will be done to their bodies. This change
of emphasis reflects a growing change in lay
attitudes toward health-care professionals,
especially physicians.
© Jamboree Education Pvt. Ltd.
Imagine flying in a hot-air balloon over the lush, Why does the text describe a trip in a hot-
green canopy of a rainforest. Through the clouds and air balloon?
mist you can barely make out the treetops and a few
A) To describe the variety of life forms in
of the birds flying among them. What lies hidden in
the rainforest.
the undergrowth? How many organisms are there,
B) To make an analogy to ocean
what do they look like, and how do they behave?
exploration.
Using a rope and bucket you blindly drag the
C) To exemplify the advanced equipment
rainforest from above hoping to ensnare some of its
that land-based biologists have at their
inhabitants or the materials that make up its
disposal.
infrastructure. But alas, with such feeble and limited
D) To show the difficulties that most
means you can learn little about the environment and
biologists encounter in extracting
life below. For years this is essentially how we have
samples from the rainforest.
studied the ocean—blindly sampling the sea with
limited and relatively ineffective methods. Even
today, with technology as advanced as it is, study of
the ocean remains a difficult and expensive task.
Whether through large-scale satellite imagery, small-
scale chemical and biological measures, or even the
collecting of fossil impressions of ancient sea
creatures, all aspects of oceanographic study require
some type of observation or sample collection, and
herein lies the problem.

© Jamboree Education Pvt. Ltd.


Imagine flying in a hot-air balloon over the lush, Why does the text describe a trip in a hot-
green canopy of a rainforest. Through the clouds and air balloon?
mist you can barely make out the treetops and a few
A) To describe the variety of life forms in
of the birds flying among them. What lies hidden in
the rainforest.
the undergrowth? How many organisms are there,
B) To make an analogy to ocean
what do they look like, and how do they behave?
exploration.
Using a rope and bucket you blindly drag the
C) To exemplify the advanced equipment
rainforest from above hoping to ensnare some of its
that land-based biologists have at their
inhabitants or the materials that make up its
disposal.
infrastructure. But alas, with such feeble and limited
D) To show the difficulties that most
means you can learn little about the environment and
biologists encounter in extracting
life below. For years this is essentially how we have
samples from the rainforest.
studied the ocean—blindly sampling the sea with
limited and relatively ineffective methods. Even
today, with technology as advanced as it is, study of
the ocean remains a difficult and expensive task.
Whether through large-scale satellite imagery, small-
scale chemical and biological measures, or even the
collecting of fossil impressions of ancient sea
creatures, all aspects of oceanographic study require
some type of observation or sample collection, and
herein lies the problem.

© Jamboree Education Pvt. Ltd.


It is likely that Web browsing strengthens Which of the following best describes the
certain brain functions, but it would be a main idea of the text?
serious mistake to look narrowly at such A) Web browsing and screen-based
technologies enhance certain cognitive
benefits and conclude that the Web is making
functions at the cost of others.
us smarter. In a science article published in B) Web browsing is mostly good for our
early 2009, prominent developmental cognitive skills.
psychologist Patricia Greenfield reviewed C) Growing dependence on screen-based
more than 40 studies of the effects of various technologies has enhanced our visual-
types of media on intelligence and learning spatial skills.
ability. She concluded that “every medium D) Excessive usage of Web browsing and
screen-based technologies is extremely
develops some cognitive skills at the expense
harmful for our brains.
of others.” Our growing use of the Net and
other screen-based technologies, she wrote,
has led to the “widespread and sophisticated
development of visual-spatial skills.” But
those gains go hand in hand with a
weakening of our capacity for the kind of
“deep processing” that underpins “mindful
knowledge acquisition, inductive analysis,
critical thinking, imagination, and reflection.”
© Jamboree Education Pvt. Ltd.
It is likely that Web browsing strengthens Which of the following best describes the
certain brain functions, but it would be a main idea of the text?
serious mistake to look narrowly at such A) Web browsing and screen-based
technologies enhance certain cognitive
benefits and conclude that the Web is making
functions at the cost of others.
us smarter. In a science article published in B) Web browsing is mostly good for our
early 2009, prominent developmental cognitive skills.
psychologist Patricia Greenfield reviewed C) Growing dependence on screen-based
more than 40 studies of the effects of various technologies has enhanced our visual-
types of media on intelligence and learning spatial skills.
ability. She concluded that “every medium D) Excessive usage of Web browsing and
screen-based technologies is extremely
develops some cognitive skills at the expense
harmful for our brains.
of others.” Our growing use of the Net and
other screen-based technologies, she wrote,
has led to the “widespread and sophisticated
development of visual-spatial skills.” But
those gains go hand in hand with a
weakening of our capacity for the kind of
“deep processing” that underpins “mindful
knowledge acquisition, inductive analysis,
critical thinking, imagination, and reflection.”
© Jamboree Education Pvt. Ltd.
HOMEWORK FOR CLASS 7

Revise all the classwork and homework of


Classes 1 to 6.

© Jamboree Education Pvt. Ltd.

You might also like